HESI study guide

Réussis tes devoirs et examens dès maintenant avec Quizwiz!

A client with cirrhosis and ascites is receiving furosemide 40 mg BID. The pharmacy provides 20 mg tablets. How many tablets should the client receive each day? [Enter numeric value only]

4 tablets 40 mg BID (BID is 2 times a day). So 40mg x 2 = 80mg/day. 80 mg day/20mg tablets available = 4 tablets a day.

A 54-year-old male client and his wife were informed this morning that he has terminal cancer. Which nursing intervention is likely to most beneficial? A. Ask her how she would like to participate in the client's care. B. Provide the wife with information about hospice C. Encourage the wife to visit after painful treatments are completed D. Refer her to support group for family members of those dying of cancer

A. Ask her how she would like to participate in the client's care. (While the client's wife may be grieving and need support, the priority for the client and client's wife is to make sure the wife feels comfortable participating in the client's care, if at all. Most people have an easier time coming to terms with the death of a loved one when they are involved in their care. (D) is a nice gesture, but will be more appropriate at a later time.)

The nurse assesses a client who has a nasal cannula delivering oxygen at 2 L/min. To assess for skin damage related to the cannula, which areas should the nurse observe? (Select all that apply). A. Tops of the ear B. Bridge of the nose C. Around the nostrils D. Over the cheeks E. Across the forehead

A. Tops of the ear C. Around the nostrils D. Over the cheeks (This is proper placement of a nasal cannula. Constant pressure from the tubing may create skin damage to the areas of skin and bony prominences the nasal cannula will be resting on.)

While reviewing the side effects of a newly prescribed medication, a 72-year-old client notes that one of the side effects is a reduction in sexual drive. Which is the best response by the nurse? A."How will this affect your present sexual activity?" B."How active is your current sex life?" C."How has your sex life changed as you have become older?" D."Tell me about your sexual needs as an older adult."

A."How will this affect your present sexual activity?"

A male client is laughing at a television program with his wife when the evening nurse enters the room. He says his foot is hurting and he would like a pain pill. How should the nurse respond? A.Ask him to rate his pain on a scale of 1 to 10. B.Encourage him to wait until bedtime so the pill can help him sleep. C.Attend to an acutely ill client's needs first because this client is laughing. D.Instruct him in the use of deep breathing exercises for pain control.

A.Ask him to rate his pain on a scale of 1 to 10. (Rationale: Obtaining a subjective estimate of the pain experience by asking the client to rate his pain helps the nurse determine which pain medication should be administered and also provides a baseline for evaluating the effectiveness of the medication. Medicating for pain should not be delayed so that it can be used as a sleep medication. Option C is judgmental. Option D should be used as an adjunct to pain medication, not instead of medication.)

An older client who had abdominal surgery 3 days earlier was given a barbiturate for sleep and is now requesting to go to the bathroom. Which action should the nurse implement? A.Assist the client to walk to the bathroom and do not leave the client alone. B.Request that the UAP assist the client onto a bedpan. C.Ask if the client needs to have a bowel movement or void. D.Assess the client's bladder to determine if the client needs to urinate.

A.Assist the client to walk to the bathroom and do not leave the client alone. (Rationale: Barbiturates cause central nervous system (CNS) depression, and individuals taking these medications are at greater risk for falls. The nurse should assist the client to the bathroom. A bedpan is not necessary as long as safety is ensured. Whether the client needs to void or have a bowel movement, option C is irrelevant in terms of meeting this client's safety needs. There is no indication that this client cannot voice her or his needs, so assessment of the bladder is not needed.)

In assisting an older adult client prepare to take a tub bath, which nursing action is most important? A.Check the bath water temperature. B.Shut the bathroom door. C.Ensure that the client has voided. D.Provide extra towels.

A.Check the bath water temperature. (Rationale: To prevent burns or excessive chilling, the nurse must check the bath water temperature. Options B, C, and D promote comfort and privacy and are important interventions but are of less priority than promoting safety.)

In taking a client's history, the nurse asks about the stool characteristics. Which description should the nurse report to the health care provider as soon as possible? A.Daily black, sticky stool B.Daily dark brown stool C.Firm brown stool every other day D.Soft light brown stool twice a day

A.Daily black, sticky stool (Rationale: Black sticky stool (melena) is a sign of gastrointestinal bleeding and should be reported to the health care provider promptly. Option C indicates constipation, which is a lesser priority. Options B and D are variations of normal.)

A client has a nursing diagnosis of Altered sleep patterns related to nocturia. Which client instruction is important for the nurse to provide? A.Decrease intake of fluids after the evening meal. B.Drink a glass of cranberry juice every day. C.Drink a glass of warm decaffeinated beverage at bedtime. D.Consult the health care provider about a sleeping pill.

A.Decrease intake of fluids after the evening meal. (Rationale: Nocturia is urination during the night. Option A is helpful to decrease the production of urine, thus decreasing the need to void at night. Option B helps prevent bladder infections. Option C may promote sleep, but the fluid will contribute to nocturia. Option D may result in urinary incontinence if the client is sedated and does not awaken to void.)

A hospitalized client has had difficulty falling asleep for two nights and is becoming irritable and restless. Which action by the nurse is best? A.Determine the client's usual bedtime routine and include these rituals in the plan of care as safety allows. B.Instruct the UAP not to wake the client under any circumstances during the night. C.Place a "Do Not Disturb" sign on the door and change assessments from every 4 to every 8 hours. D.Encourage the client to avoid pain medication during the day, which might increase daytime napping.

A.Determine the client's usual bedtime routine and include these rituals in the plan of care as safety allows. (Rationale: Including habitual rituals that do not interfere with the client's care or safety may allow the client to go to sleep faster and increase the quality of care. Options B, C, and D decrease the client's standard of care and compromise safety.)

The nurse is using the Glasgow Coma Scale to perform a neurologic assessment. A comatose client winces and pulls away from a painful stimulus. Which action should the nurse take next? A.Document that the client responds to painful stimulus. B.Observe the client's response to verbal stimulation. C.Place the client on seizure precautions for 24 hours. D.Report decorticate posturing to the health care provider

A.Document that the client responds to painful stimulus. (Rationale: The client has demonstrated a purposeful response to pain, which should be documented as such. Response to painful stimulus is assessed after response to verbal stimulus, not before. There is no indication for placing the client on seizure precautions. Reporting decorticate posturing to the health care provider is nonpurposeful movement.)

While conducting an intake assessment of an adult male at a community mental health clinic, the nurse notes that his affect is flat, he responds to questions with short answers, and he reports problems with sleeping. He reports that his life partner recently died from pneumonia. Which action is most important for the nurse to implement? A.Encourage the client to see the clinic's grief counselor. B.Determine if the client has a family history of suicide attempts. C.Inquire about whether the life partner was suffering from AIDS. D.Consult with the health care provider about the client's need for antidepressant medications.:

A.Encourage the client to see the clinic's grief counselor.

The nurse is aware that malnutrition is a common problem among clients served by a community health clinic for the homeless. Which laboratory value is the most reliable indicator of chronic protein malnutrition? A.Low serum albumin level B.Low serum transferrin level C.High hemoglobin level D.High cholesterol level

A.Low serum albumin level (Rationale: Long-term protein deficiency is required to cause significantly lowered serum albumin levels. Albumin is made by the liver only when adequate amounts of amino acids (from protein breakdown) are available. Albumin has a long half-life, so acute protein loss does not significantly alter serum levels. Option B is a serum protein with a half-life of only 8 to 10 days, so it will drop with an acute protein deficiency. Options C and D are not clinical measures of protein malnutrition.)

Based on the nursing diagnosis of risk for infection, which intervention is best for the nurse to implement when providing care for an older incontinent client? A.Maintain standard precautions. B.Initiate contact isolation measures. C.Insert an indwelling urinary catheter. D.Instruct client in the use of adult diapers.

A.Maintain standard precautions. (Rationale: The best action to decrease the risk of infection in vulnerable clients is handwashing. Option B is not necessary unless the client has an infection. Option C increases the risk of infection. Option D does not reduce the risk of infection.)

By rolling contaminated gloves inside-out, the nurse is affecting which step in the chain of infection? A.Mode of transmission B.Portal of entry C.Reservoir D.Portal of exit

A.Mode of transmission (Rationale: The contaminated gloves serve as the mode of transmission from the portal of exit of the reservoir to a portal of entry.)

The nurse-manager of a skilled nursing (chronic care) unit is instructing UAPs on ways to prevent complications of immobility. Which intervention should be included in this instruction? A.Perform range-of-motion exercises to prevent contractures. B.Decrease the client's fluid intake to prevent diarrhea. C.Massage the client's legs to reduce embolism occurrence. D.Turn the client from side to back every shift.

A.Perform range-of-motion exercises to prevent contractures. (Rationale: Performing range-of-motion exercises is beneficial in reducing contractures around joints. Options B, C, and D are all potentially harmful practices that place the immobile client at risk of complications.)

The nurse prepares to insert a nasogastric tube in a client with hyperemesis who is awake and alert. Which intervention(s) is(are) correct? (Select all that apply.) A.Place the client in a high Fowler position. B.Help the client assume a left side-lying position. C.Measure the tube from the tip of the nose to the umbilicus. D.Instruct the client to swallow after the tube has passed the pharynx. E.Assist the client in extending the neck back so the tube may enter the larynx.

A.Place the client in a high Fowler position. D.Instruct the client to swallow after the tube has passed the pharynx.

Which step(s) should the nurse take when administering ear drops to an adult client? (Select all that apply.) A.Place the client in a side-lying position. B.Pull the auricle upward and outward. C.Hold the dropper 6 cm above the ear canal. D.Place a cotton ball into the inner canal. E.Pull the auricle down and back.

A.Place the client in a side-lying position. B.Pull the auricle upward and outward. (Rationale: The correct answers (A and B) are the appropriate administration of ear drops. The dropper should be held 1 cm (1⁄2 inch) above the ear canal (C). A cotton ball should be placed in the outermost canal (D). The auricle is pulled down and back for a child younger than 3 years of age, but not an adult (E).)

A nurse is working in an occupational health clinic when an employee walks in and states that he was struck by lightning while working in a truck bed. The client is alert but reports feeling faint. Which assessment will the nurse perform first? A.Pulse characteristics B.Open airway C.Entrance and exit wounds D.Cervical spine injury

A.Pulse characteristics (Rationale: Lightning is a jolt of electrical current and can produce a "natural" defibrillation, so assessment of the pulse rate and regularity is a priority. Because the client is talking, he has an open airway, so that assessment is not necessary. Assessing for options C and D should occur after assessing for adequate circulation.)

When emptying 350 mL of pale yellow urine from a client's urinal, the nurse notes that this is the first time the client has voided in 4 hours. Which action should the nurse take next? A.Record the amount on the client's fluid output record. B.Encourage the client to increase oral fluid intake. C.Notify the health care provider of the findings. D.Palpate the client's bladder for distention.:

A.Record the amount on the client's fluid output record.

Which client is most likely to be at risk for spiritual distress? A.Roman Catholic woman considering an abortion B.Jewish man considering hospice care for his wife C.Seventh-Day Adventist who needs a blood transfusion D.Muslim man who needs a total knee replacement

A.Roman Catholic woman considering an abortion

Which nonverbal action should the nurse implement to demonstrate active listening? A.Sit facing the client. B.Cross arms and legs. C.Avoid eye contact. D.Lean back in the chair.

A.Sit facing the client.

The nurse is obtaining a lie-sit-stand blood pressure reading on a client. Which action is most important for the nurse to implement? A.Stay with the client while the client is standing. B.Record the findings on the graphic sheet in the chart. C.Keep the blood pressure cuff on the same arm. D.Record changes in the client's pulse rate

A.Stay with the client while the client is standing. (Rationale: Although all these measures are important, option A is most important because it helps ensure client safety. Option B is necessary but does not have the priority of option A. Options C and D are important measures to ensure accuracy of the recording but are of less importance than providing client safety.)

The nurse retrieves hydromorphone 4mg/mL from the Pyxis MedStation, an automated dispensing system, for a client who is receiving hydromorphone 3 mg IM 6 hours PRN for severe pain. How many mL should the nurse administer to the client? (Enter the numerical value only. If rounding is required, round to the nearest tenth)

Ans: 0.8 4mg/1 mL = 3mg/ X mL : 3mg/4mg = 0.75 (READ THE ROUNDING CAREFULLY, here is says to the nearest tenth. Therefore, the answer is 0.8 mL).

The nurse is counting a client's respiratory rate. During a 30-second interval, the nurse counts six respirations and the client coughs three times. In repeating the count for a second 30-second interval, the nurse counts eight respirations. Which respiratory rate should the nurse document? A.14 B. 16 C.17 D.28

B. 16 (Rationale: The most accurate respiratory rate is the second count obtained by the nurse, which was not interrupted by coughing. Because it was counted for 30 seconds, the rate should be doubled. Options A, C, and D are inaccurate recordings.)

The nurse is instructing a client in the proper use of a metered-dose inhaler. Which instruction should the nurse provide the client to ensure the optimal benefits from the drug? A."Fill your lungs with air through your mouth and then compress the inhaler." B."Compress the inhaler while slowly breathing in through your mouth." C."Compress the inhaler while inhaling quickly through your nose." D."Exhale completely after compressing the inhaler and then inhale."

B."Compress the inhaler while slowly breathing in through your mouth." (Rationale: The medication should be inhaled through the mouth simultaneously with compression of the inhaler. This will facilitate the desired destination of the aerosol medication deep in the lungs for an optimal bronchodilation effect. Options A, C, and D do not allow for deep lung penetration.)

Ten minutes after signing an operative permit for a fractured hip, an older client states, "The aliens will be coming to get me soon!" and falls asleep. Which action should the nurse implement next? A.Make the client comfortable and allow the client to sleep. B.Assess the client's neurologic status. C.Notify the surgeon about the comment. D.Ask the client's family to co-sign the operative permit

B.Assess the client's neurologic status.

The nurse is instructing a client with cholecystitis regarding diet choices. Which meal best meets the dietary needs of this client? A.Steak, baked beans, and a salad B.Broiled fish, green beans, and an apple C.Pork chops, macaroni and cheese, and grapes D.Avocado salad, milk, and angel food cake

B.Broiled fish, green beans, and an apple (Rationale: Clients with cholecystitis (inflammation of the gallbladder) should follow a low- fat diet, such as option B. Option A is a high-protein diet, and options C and D contain high- fat foods, which are contraindicated for this client.)

A 65-year-old client who attends an adult daycare program and is wheelchair-mobile has redness in the sacral area. Which instruction is most important for the nurse to provide? A.Take a vitamin supplement tablet once a day. B.Change positions in the chair at least every hour. C.Increase daily intake of water or other oral fluids. D.Purchase a newer model wheelchair.

B.Change positions in the chair at least every hour. (Rationale: The most important teaching is to change positions frequently because pressure is the most significant factor related to the development of pressure ulcers. Increased vitamin and fluid intake may also be beneficial and promote healing and reduce further risk. Option D is an intervention of last resort because this will be very expensive for the client.)

The nurse assesses a 2-year-old who is admitted for dehydration and finds that the peripheral IV rate by gravity has slowed, even though the venous access site is healthy. What should the nurse do next? A.Apply a warm compress proximal to the site. B.Check for kinks in the tubing and raise the IV pole. C.Adjust the tape that stabilizes the needle. D.Flush with normal saline and recount the drop rate.

B.Check for kinks in the tubing and raise the IV pole. (Rationale: The nurse should first check the tubing and height of the bag on the IV pole, which are common factors that may slow the rate. Gravity infusion rates are influenced by the height of the bag, tubing clamp closure or kinks, needle size or position, fluid viscosity, client blood pressure (crying in the pediatric client), and infiltration. Venospasm can slow the rate and often responds to warmth over the vessel, but the nurse should first adjust the IV pole height. The nurse may need to adjust the stabilizing tape on a positional needle or flush the venous access with normal saline, but less invasive actions should be implemented first.)

A nurse is assigned to care for a close friend in the hospital setting. Which action should the nurse take first when given the assignment? A.Notify the friend that all medical information will be kept confidential. B.Explain the relationship to the charge nurse and ask for reassignment. C.Approach the client and ask if the assignment is uncomfortable. D.Accept the assignment but protect the client's confidentiality.

B.Explain the relationship to the charge nurse and ask for reassignment. (Rationale: Caring for a close friend can violate boundaries for nurses and should be avoided when possible (B). If the assignment is unavoidable (there are no other nurses to care for the client) then C, A, and D should be addressed.)

When turning an immobile bedridden client without assistance, which action by the nurse best ensures client safety? A.Securely grasp the client's arm and leg. B.Put bed rails up on the side of bed opposite from the nurse. C.Correctly position and use a turn sheet. D.Lower the head of the client's bed slowly.

B.Put bed rails up on the side of bed opposite from the nurse.

During evacuation of a group of clients from a medical unit because of a fire, the nurse observes an ambulatory client walking alone toward the stairway at the end of the hall. Which action should the nurse take? A.Assign an unlicensed assistive personnel to transport the client via a wheelchair. B.Remind the client to walk carefully down the stairs until reaching a lower floor. C.Ask the client to help by assisting a wheelchair-bound client to a nearby elevator. D.Open the closest fire doors so that ambulatory clients can evacuate more rapidly.

B.Remind the client to walk carefully down the stairs until reaching a lower floor. (Rationale: During evacuation of a unit because of fire, ambulatory clients should be evacuated via the stairway if at all possible and reminded to walk carefully. Ambulatory clients do not require the assistance of a wheelchair to be evacuated. Elevators should not be used during a fire, and fire doors should be kept closed to help contain the fire.)

The nurse selects the best site for insertion of an IV catheter in the client's right arm. Which documentation should the nurse use to identify placement of the IV access? A.Left brachial vein B.Right cephalic vein C.Dorsal side of the right wrist D.Right upper extremity

B.Right cephalic vein (Rationale: The cephalic vein is large and superficial and identifies the anatomic name of the vein that is accessed, which should be included in the documentation. The basilic vein of the arm is used for IV access, not the brachial vein, which is too deep to be accessed for IV infusion. Although veins on the dorsal side of the right wrist are visible, they are fragile and using them would be painful, so they are not recommended for IV access. Option D is not specific enough for documenting the location of the IV access.)

After a needle stick occurs while removing the cap from a sterile needle, which action should the nurse implement? A.Complete an incident report. B.Select another sterile needle. C.Disinfect the needle with an alcohol swab. D.Notify the supervisor of the department immediately.

B.Select another sterile needle. (Rationale: After a needle stick, the needle is considered used, so the nurse should discard it and select another needle. Because the needle was sterile when the nurse was stuck and the needle was not in contact with any other person's body fluids, the nurse does not need to complete an incident report or notify the occupational health nurse. Disinfecting a needle with an alcohol swab is not in accordance with standards for safe practice and infection control.)

Which action should the nurse implement when providing wound care instructions to a client who does not speak English? A.Ask an interpreter to provide wound care instructions. B.Speak directly to the client, with an interpreter translating. C.Request the accompanying family member to translate. D.Instruct a bilingual employee to read the instructions.

B.Speak directly to the client, with an interpreter translating.

The nurse teaches the use of a gait belt to a male caregiver whose wife has right-sided weakness and needs assistance with ambulation. The caregiver performs a return demonstration of the skill. Which observation indicates that the caregiver has learned how to perform this procedure correctly? A.Standing on his wife's strong side, the caregiver is ready to hold the gait belt if any evidence of weakness is observed. B.Standing on his wife's weak side, the caregiver provides security by holding the gait belt from the back. C.Standing behind his wife, the caregiver provides balance by holding both sides of the gait belt. D.Standing slightly in front and to the right of his wife, the caregiver guides her forward by gently pulling on the gait belt.

B.Standing on his wife's weak side, the caregiver provides security by holding the gait belt from the back. (Rationale: His wife is most likely to lean toward the weak side and needs extra support on that side and from the back to prevent falling. Options A, C, and D provide less security for her.)

The nurse is assessing several clients prior to surgery. Which factor in a client's history poses the greatest threat for complications to occur during surgery? A.Taking birth control pills for the past 2 years B.Taking anticoagulants for the past year C.Recently completing antibiotic therapy D.Having taken laxatives PRN for the last 6 months

B.Taking anticoagulants for the past year (Anticoagulants increase the risk for bleeding during surgery, which can pose a threat for the development of surgical complications. The health care provider should be informed that the client is taking these drugs. Although clients who take birth control pills may be more susceptible to the development of thrombi, such problems usually occur postoperatively. A client with option C or D is at less of a surgical risk than with option B.)

A seriously ill female client tells the nurse, "I am so tired and in so much pain! Please help me to die." Which is the best response for the nurse to provide? A.Administer the prescribed maximum dose of pain medication B.Talk with the client about her feelings related to her own death. C.Collaborate with the health care provider about initiating antidepressant therapy. D.Refer the client to the ethics committee of her local health care facility

B.Talk with the client about her feelings related to her own death. (Rationale: The nurse should first assess the client's feelings about her death and determine the extent to which this statement expresses her true feelings. The client may need additional pain management, but further assessment is needed before implementing option A. Options C and D are both premature interventions and should not be implemented until further assessment is obtained.)

The nurse observes a UAP taking a client's blood pressure in the lower extremity. Which observation of this procedure requires the nurse's intervention? A.The cuff wraps around the girth of the leg. B.The UAP auscultates the popliteal pulse with the cuff on the lower leg. C.The client is placed in a prone position. D.The systolic reading is 20 mm Hg higher than the blood pressure in the client's arm.

B.The UAP auscultates the popliteal pulse with the cuff on the lower leg. (Rationale: When obtaining the blood pressure in the lower extremities, the popliteal pulse is the site for auscultation when the blood pressure cuff is applied around the thigh. The nurse should intervene with the UAP who has applied the cuff on the lower leg. Option A ensures an accurate assessment, and option C provides the best access to the artery. Systolic pressure in the popliteal artery is usually 10 to 40 mm Hg higher than in the brachial artery.)

The health care provider has changed a client's prescription from the PO to the IV route of administration. The nurse should anticipate which change in the pharmacokinetic properties of the medication? A.The client will experience increased tolerance to the drug's effects and may need a higher dose. B.The onset of action of the drug will occur more rapidly, resulting in a more rapid effect. C.The medication will be more highly protein-bound, increasing the duration of action. D.The therapeutic index will be increased, placing the client at greater risk for toxicity.

B.The onset of action of the drug will occur more rapidly, resulting in a more rapid effect. (Rationale: Because the absorptive process is eliminated when medications are administered via the IV route, the onset of action is more rapid, resulting in a more immediate effect. Drug tolerance, protein binding, and the drug's therapeutic index are not affected by the change in route from PO to IV. In addition, an increased therapeutic index reduces the risk of drug toxicity.)

The nurse identifies a potential for infection in a client with partial-thickness (second- degree) and full-thickness (third-degree) burns. What intervention has the highest priority in decreasing the client's risk of infection? A.Administration of plasma expanders B.Use of careful handwashing technique C.Application of a topical antibacterial cream D.Limiting visitors to the client with burns

B.Use of careful handwashing technique (Rationale: Careful handwashing technique is the single most effective intervention for the prevention of contamination to all clients. Option A reverses the hypovolemia that initially accompanies burn trauma but is not related to decreasing the proliferation of infective organisms. Options C and D are recommended by various burn centers as possible ways to reduce the chance of infection. Option B is a proven technique to prevent infection.)

When assisting a client from the bed to a chair, which procedure is best for the nurse to follow? A.Place the chair parallel to the bed, with its back toward the head of the bed and assist the client in moving to the chair. B.With the nurse's feet spread apart and knees aligned with the client's knees, stand and pivot the client into the chair. C.Assist the client to a standing position by gently lifting upward, underneath the axillae. D.Stand beside the client, place the client's arms around the nurse's neck, and gently move the client to the chair.

B.With the nurse's feet spread apart and knees aligned with the client's knees, stand and pivot the client into the chair.

Which assessment data would provide the most accurate determination of proper placement of a nasogastric tube? A) Aspirating gastric contents to assure a pH value of 4 or less. B) Hearing air pass in the stomach after injecting air into the tubing. C) Examining a chest x-ray obtained after the tubing was inserted. D) Checking the remaining length of tubing to ensure that the correct length was inserted.

C) Examining a chest x-ray obtained after the tubing was inserted. (After placing an NG-tube, the placement of the tube is confirmed via x-ray since it is the most accurate way to ensure the tube has not been placed in the lungs, which would pose an aspiration risk.)

The nurse explains to an older adult male the procedure for collecting a 24-hour urine specimen for creatinine clearance. Which action is most important for the nurse to include in their care plan for the shift? A. Assess the client for confusion and reteach the procedure B. Check the urine for color and texture C. Empty the urinal contents into the 24-hour collection container D. Discard the contents of the urinal

C. Empty the urinal contents into the 24-hour collection container (An "older adult male" in the question may imply that the patient may have an altered mental status or be demented. While suggesting, it is not directly stated, therefore (A) is inappropriate. (B) is incorrect because the lab will be assessing the collection specimen after the test is complete. (C) is correct because the nurse should first discard the first specimen, then begin to collect and record the time the first urine specimen was collected. It is important to have strict documentation for output, and to collect every urine specimen within that 24 hour period, otherwise the test must be restarted. (D) defeats the purpose of the 24-hour urine collection test.)

The father of an 11-year-old client reports to the nurse that the client has been "wetting the bed" since the passing of his mother and is concerned. Which action is most important for the nurse to enact? A. Reassure the father that it is normal for a pre-teen to wet the bed during puberty. B. Inform the father that nocturnal emissions are abnormal and his son is developmentally delayed C. Inform the father that it is most important to let the son know that nocturnal emissions are normal after trauma D. Refer the father and the client to a psychologist

C. Inform the father that it is most important to let the son know that nocturnal emissions are normal after trauma (It is common for adolescents to regress in their biological progression after experiencing a severe trauma, like losing a parent, sibling, or friend. While uncomfortable for the adolescent and parent, it is nothing to be concerned for. Often times, as the patient grieves or comes to terms with the trauma, the nocturnal emissions will cease.)

While changing a client's post-operative dressing, the nurse observes a red and swollen wound with a moderate amount of yellow and green drainage and a foul odor. Given there is a positive MRSA, which is the most important action for the nurse to take? A. Force oral fluids B. Request a nutrition consult C. Initiate contact precautions D. Limit visitors to immediate family only

C. Initiate contact precautions (MRSA is a type of antibiotic resistant bacteria and a patient with this should be placed on contact precautions. (A) oral fluids will not help rid the patient of the infection. (B) nor nutrition. (D) limiting visitors to immediate family is not necessary as anyone is at risk for contracting MRSA from an infected wound.)

A client who has a body mass index (BMI) of 30 is requesting information on the initial approach to a weight loss plan. Which action should the nurse recommend first? A. Plan low carbohydrate and high protein meals B. Engage in strenuous activity for an hour daily C. Keep a record of food and drinks consumed daily D. Participated in a group exercise class 3 times a week

C. Keep a record of food and drinks consumed daily (BMI of 30 indicates the patient is obese. (A) While a good step, it is not what should be completed first. (B) While a good step, it is not what should be completed first. (C) The best thing to recommend is to have the patient keep a food journal to be able to go back and track their calorie intake; it may be helpful when meal planning or creating a workout routine plan. (D) Would be appropriate later.)

The nurse is teaching an obese client, newly diagnosed with arteriosclerosis, about reducing the risk of a heart attack or stroke. Which health promotion brochure is most important for the nurse to provide to this client? A."Monitoring Your Blood Pressure at Home" B."Smoking Cessation as a Lifelong Commitment" C."Decreasing Cholesterol Levels Through Diet" D."Stress Management for a Healthier You"

C."Decreasing Cholesterol Levels Through Diet" (Rationale: A health promotion brochure about decreasing cholesterol is most important to provide this client, because the most significant risk factor contributing to development of arteriosclerosis is excess dietary fat, particularly saturated fat and cholesterol. Option A does not address the underlying causes of arteriosclerosis. Options B and D are also important factors for reversing arteriosclerosis but are not as important as lowering cholesterol.)

One week after being told that she has terminal cancer with a life expectancy of 3 weeks, a female client tells the nurse, "I think I will plan a big party for all my friends." How should the nurse respond? A."You may not have enough energy before long to hold a big party." B."Do you mean to say that you want to plan your funeral and wake?" C."Planning a party and thinking about all your friends sounds like fun." D."You should be thinking about spending your last days with your family."

C."Planning a party and thinking about all your friends sounds like fun." (Rationale: Setting goals that bring pleasure is appropriate and should be encouraged by the nurse as long as the nurse does not perpetuate a client's denial. Option A is a negative response, implying that the client should not plan a party. Option B puts words in the client's mouth that may not be accurate. The nurse should support the client's goals rather than telling the client how to spend her time.)

An older adult who recently began self-administration of insulin calls the nurse daily to review the steps that should be taken when giving an injection. The nurse has assessed the client's skills during two previous office visits and knows that the client is capable of giving the daily injection. Which response by the nurse is likely to be most helpful in encouraging the client to assume total responsibility for the daily injections? A."I know you are capable of giving yourself the insulin." B."Giving yourself the injection seems to make you nervous." C."When I watched you give yourself the injection, you did it correctly." D."Tell me what you want me to do to help you give yourself the injection at home."

C."When I watched you give yourself the injection, you did it correctly." Rationale: The nurse needs to focus on the client's positive behaviors, so focusing on the client's demonstrated ability to self-administer the injection is likely to reinforce his level of competence without sounding punitive. Option A does not focus on the specific behaviors related to giving the injection and could be interpreted as punitive. Option B uses reflective dialogue to assess the client's feelings, but telling the client that he is nervous may serve as a negative reinforcement of this behavior. Option D reinforces the client's dependence on the nurse.

A community hospital is opening a mental health services department. Which document should the nurse use to develop the unit's nursing guidelines? A.Americans with Disabilities Act of 1990 B.ANA Code of Ethics with Interpretative Statements C.ANA's Scope and Standards of Nursing Practice D.Patient's Bill of Rights of 1990

C.ANA's Scope and Standards of Nursing Practice (Rationale: The ANA Scope of Standards of Practice for Psychiatric-Mental Health Nursing serves to direct the philosophy and standards of psychiatric nursing practice. Options A and D define the client's rights. Option B provides ethical guidelines for nursing.)

The nurse is preparing to administer 10 mL of liquid potassium chloride through a feeding tube, followed by 10 mL of liquid acetaminophen. Which action should the nurse include in this procedure? A.Dilute each of the medications with sterile water prior to administration. B.Mix the medications in one syringe before opening the feeding tube. C.Administer water between the doses of the two liquid medications. D.Withdraw any fluid from the tube before instilling each medication.

C.Administer water between the doses of the two liquid medications. (Rationale: Water should be instilled into the feeding tube between administering the two medications to maintain the patency of the feeding tube and ensure that the total dose of medication enters the stomach and does not remain in the tube. These liquid medications do not need to be diluted when administered via a feeding tube and should be administered separately, with water instilled between each medication.)

A client in a long-term care facility reports to the nurse that he has not had a bowel movement in 2 days. Which intervention should the nurse implement first? A.Instruct the caregiver to offer a glass of warm prune juice at mealtimes. B.Notify the health care provider and request a prescription for a large-volume enema. C.Assess the client's medical record to determine the client's normal bowel pattern. D.Instruct the caregiver to increase the client's fluids to five 8-ounce glasses per day.

C.Assess the client's medical record to determine the client's normal bowel pattern. (Rationale: This client may not routinely have a daily bowel movement, so the nurse should first assess this client's normal bowel habits before attempting any intervention. Option A, B, or D may then be implemented, if warranted.)

After the nurse tells an older client that an IV line needs to be inserted, the client becomes very apprehensive, loudly verbalizing a dislike for all health care providers and nurses. How should the nurse respond? A.Ask the client to remain quiet so the procedure can be performed safely. B.Concentrate on completing the insertion as efficiently as possible. C.Calmly reassure the client that the discomfort will be temporary. D.Tell the client a joke as a means of distraction from the procedure

C.Calmly reassure the client that the discomfort will be temporary. (Rationale: The nurse should respond with a calm demeanor to help reduce the client's apprehension. After responding calmly to the client's apprehension, the nurse may implement to ensure safe completion of the procedure.)

A female client with frequent urinary tract infections (UTIs) asks the nurse to explain her friend's advice about drinking a glass of juice daily to prevent future UTIs. Which response is best for the nurse to provide? A.Orange juice has vitamin C that deters bacterial growth. B.Apple juice is the most useful in acidifying the urine. C.Cranberry juice stops pathogens' adherence to the bladder. D.Grapefruit juice increases absorption of most antibiotics.

C.Cranberry juice stops pathogens' adherence to the bladder. (Rationale: Cranberry juice maintains urinary tract health by reducing the adherence of Escherichia coli bacteria to cells within the bladder. Options A, B, and D have not been shown to be as effective as cranberry juice in preventing UTIs.)

When taking a client's blood pressure, the nurse is unable to distinguish the point at which the first sound was heard. Which is the best action for the nurse to take? A.Deflate the cuff completely and immediately reattempt the reading. B.Reinflate the cuff completely and leave it inflated for 90 to 110 seconds before taking the second reading. C.Deflate the cuff to zero and wait 30 to 60 seconds before reattempting the reading. D.Document the exact level visualized on the sphygmomanometer where the first fluctuation was seen.:

C.Deflate the cuff to zero and wait 30 to 60 seconds before reattempting the reading.

The nurse who is preparing to give an adolescent client a prescribed antipsychotic medication notes that parental consent has not been obtained. Which action should the nurse take? A.Review the chart for a signed consent for hospitalization. B.Get the health care provider's permission to give the medication. C.Do not give the medication and document the reason. D.Complete an incident report and notify the parents.:

C.Do not give the medication and document the reason. (Rationale: The nurse should not give the medication and should document the reason because the client is a minor and needs a guardian's permission to receive medications. Permission to give medications is not granted by a signed hospital consent or a health care provider's permission, unless conditions are met to justify coerced treatment. Option D is not necessary unless the medication had previously been administered.)

Which instruction is most important for the nurse to include when teaching a client with limited mobility strategies to prevent venous thrombosis? A.Perform cough and deep breathing exercises hourly. B.Turn from side to side in bed at least every 2 hours. C.Dorsiflex and plantarflex the feet 10 times each hour. D.Drink approximately 4 ounces of water every hour

C.Dorsiflex and plantarflex the feet 10 times each hour.

In completing a client's preoperative routine, the nurse finds that the operative permit is not signed. The client begins to ask more questions about the surgical procedure. Which action should the nurse take next? A.Witness the client's signature to the permit. B.Answer the client's questions about the surgery. C.Inform the surgeon that the operative permit is not signed and the client has questions about the surgery. D.Reassure the client that the surgeon will answer any questions before the anesthesia is administered.

C.Inform the surgeon that the operative permit is not signed and the client has questions about the surgery. (Rationale: The surgeon should be informed immediately that the permit is not signed. It is the surgeon's responsibility to explain the procedure to the client and obtain the client's signature on the permit. Although the nurse can witness an operative permit, the procedure must first be explained by the health care provider or surgeon, including answering the client's questions. The client's questions should be addressed before the permit is signed.)

Urinary catheterization is prescribed for a postoperative female client who has been unable to void for 8 hours. The nurse inserts the catheter, but no urine is seen in the tubing. Which action will the nurse take next? A.Clamp the catheter and recheck it in 60 minutes. B.Pull the catheter back 3 inches and redirect upward. C.Leave the catheter in place and reattempt with another catheter. D.Notify the health care provider of a possible obstruction.

C.Leave the catheter in place and reattempt with another catheter.

When bathing an uncircumcised boy older than 3 years, which action should the nurse take? A.Remind the child to clean his genital area. B.Defer perineal care because of the child's age. C.Retract the foreskin gently to cleanse the penis. D.Ask the parents why the child is not circumcised.

C.Retract the foreskin gently to cleanse the penis.

A nurse stops at a motor vehicle collision site to render aid until the emergency personnel arrive and applies pressure to a groin wound that is bleeding profusely. Later the client has to have the leg amputated and sues the nurse for malpractice. Which is the most likely outcome of this lawsuit? A.The Patient's Bill of Rights protects clients from malicious intents, so the nurse could lose the case. B.The lawsuit may be settled out of court, but the nurse's license is likely to be revoked. C.There will be no judgment against the nurse, whose actions were protected under the Good Samaritan Act. D.The client will win because the four elements of negligence (duty, breach, causation, and damages) can be proved.

C.There will be no judgment against the nurse, whose actions were protected under the Good Samaritan Act. (Rationale: The Good Samaritan Act protects health care professionals who practice in good faith and provide reasonable care from malpractice claims, regardless of the client outcome. Although the Patient's Bill of Rights protects clients, this nurse is protected by the Good Samaritan Act. The state Board of Nursing has no reason to revoke a registered nurse's license unless there was evidence that actions taken in the emergency were not done in good faith or that reasonable care was not provided. All four elements of malpractice were not shown.)

The nurse plans to assist a male client out of bed for the first time since his surgery yesterday. His wife objects and tells the nurse to get out of the room because her husband is too ill to get out of bed. A. Administer nasal oxygen at a rate of 5 L/min B. Help the client to lie back down in the bed C. Quickly pivot the client to the chair and elevate the legs D. Check the client's blood pressure and pulse deficit

D. Check the client's blood pressure and pulse deficit ((A) is incorrect because it does not say that the patient has a decrease in O2 stats. (B) is incorrect because you assume the patient is already laying down and you have a task at hand. (C) MOVE SLOW after surgery! Never quickly move a patient. You could injury yourself and the client. (D) is correct; make sure they are physiologically stable first before moving them.)

The nurse is reviewing the signed operative consent with a client who is admitted for the removal of a lipoma on the left leg. The client states that the consent form should say the removal of a lipoma on the right leg. Which intervention should the nurse implement? A. Notify the OR staff of the client's confusion B. Have the client sign a new surgical consent C. Add the additional information to the consent D. Inform the surgeon about the client's concern

D. Inform the surgeon about the client's concern (If there are any discrepencies or concerns from the patient prior to signing the surgery consent, then the RN needs to call the surgeon to come speak to the patient to clear any confusion. (A) this client is not confused, never assume they are. (B) You will do this after the proper leg is discussed with the surgeon. (C) never add information to a consent! That is not your job and is only done with the surgeon's knowledge.)

The nurse is administering the 0900 medications to a client who was admitted during the night. Which client statement indicates that the nurse should further assess the medication order? A."At home I take my pills at 8:00 am." B."It costs a lot of money to buy all of these pills." C."I get so tired of taking pills every day." D."This is a new pill I have never taken before."

D."This is a new pill I have never taken before." (Rationale: The client's recognition of a "new" pill requires further assessment to verify that the medication is correct, if it is a new prescription or a different manufacturer, or if the client needs further instruction. The time difference may not be as significant in terms of its effect, but this should be explained. Although comments about cost should be considered when developing a discharge plan, option D is a higher priority. The client's feelings C should be acknowledged, but observation of the five rights of medication administration is most essential.)

The nurse is teaching a client how to perform progressive muscle relaxation techniques to relieve insomnia. A week later the client reports that he is still unable to sleep, despite following the same routine every night. Which action should the nurse take first? A.Instruct the client to add regular exercise as a daily routine. B.Determine if the client has been keeping a sleep diary. C.Encourage the client to continue the routine until sleep is achieved. D.Ask the client to describe the routine he is currently following.

D.Ask the client to describe the routine he is currently following. (Rationale: The nurse should first evaluate whether the client has been adhering to the original instructions. A verbal report of the client's routine will provide more specific information than the client's written diary. The nurse can then determine which changes need to be made. The routine practiced by the client is clearly unsuccessful, so encouragement alone is insufficient.)

During a routine assessment, an obese 50-year-old female client expresses concern about her sexual relationship with her husband. Which is the best response by the nurse? A.Reassure the client that many obese people have concerns about sex. B.Remind the client that sexual relationships need not be affected by obesity. C.Determine the frequency of sexual intercourse. D.Ask the client to talk about specific concerns

D.Ask the client to talk about specific concerns. (Rationale: Option D provides an opportunity for the client to verbalize her concerns and provides the nurse with more assessment data. Options A and B may not be related to her current concern, assume that obesity is the problem, and are communication blocks. Option C may be appropriate after discussing the concerns she is having.)

A client's blood pressure reading is 156/94 mm Hg. Which action should the nurse take first? A.Tell the client that the blood pressure is high and that the reading needs to be verified by another nurse. B.Contact the health care provider to report the reading and obtain a prescription for an antihypertensive medication. C.Replace the cuff with a larger one to ensure an ample fit for the client to increase arm comfort. D.Compare the current reading with the client's previously documented blood pressure readings.

D.Compare the current reading with the client's previously documented blood pressure readings. (Rationale: Comparing this reading with previous readings will provide information about what is normal for this client; this action should be taken first. Option A might unnecessarily alarm the client. Option B is premature. Further assessment is needed to determine if the reading is abnormal for this client. Option C could falsely decrease the reading and is not the correct procedure for obtaining a blood pressure reading.)

The nurse transcribes the postoperative prescriptions for a client who returns to the unit following surgery and notes that an antihypertensive medication that was prescribed preoperatively is not listed. Which action should the nurse take? A.Consult with the pharmacist about the need to continue the medication. B.Administer the antihypertensive medication as prescribed preoperatively. C.Withhold the medication until the client is fully alert and vital signs are stable. D.Contact the health care provider to renew the prescription for the medication.

D.Contact the health care provider to renew the prescription for the medication. (Rationale: Medications prescribed preoperatively must be renewed postoperatively, so the nurse should contact the health care provider if the antihypertensive medication is not included in the postoperative prescriptions. The pharmacist does not prescribe medications or renew prescriptions. The nurse must have a current prescription before administering any medications.)

During a clinic visit, the mother of a 7-year-old reports to the nurse that her child is often awake until midnight playing and is then very difficult to awaken in the morning for school. Which assessment data should the nurse obtain in response to the mother's report? A.The occurrence of any episodes of sleep apnea B.The child's blood pressure, pulse, and respirations C.Length of rapid eye movement (REM) sleep that the child is experiencing D.Description of the family's home environment

D.Description of the family's home environment (Rationale: School-age children often resist bedtime. The nurse should begin by assessing the environment of the home to determine factors that may not be conducive to the establishment of bedtime rituals that promote sleep. Option A often causes daytime fatigue rather than resistance to going to sleep. Option B is unlikely to provide useful data. The nurse cannot determine option C.)

The nurse determines that a postoperative client's respiratory rate has increased from 18 to 24 breaths/min. Based on this assessment finding, which intervention is most important for the nurse to implement? A.Encourage the client to increase ambulation in the room. B.Offer the client a high-carbohydrate snack for energy. C.Force fluids to thin the client's pulmonary secretions. D.Determine if pain is causing the client's tachypnea.

D.Determine if pain is causing the client's tachypnea. (Rationale: Pain, anxiety, and increasing fluid accumulation in the lungs can cause tachypnea (increased respiratory rate). Encouraging the client to increase ambulation when the respiratory rate is rising above normal limits puts the client at risk for further oxygen desaturation. Option B can increase the client's carbon metabolism, so an alternative source of energy, such as Pulmocare liquid supplement, should be offered instead. Option C could increase respiratory congestion in a client with a poorly functioning cardiopulmonary system, placing the client at risk of fluid overload.)

When performing sterile wound care in the acute care setting, the nurse obtains a bottle of normal saline from the bedside table that is labeled "opened" and dated 48 hours prior to the current date. Which is the best action for the nurse to take? A.Use the normal saline solution once more and then discard. B.Obtain a new sterile syringe to draw up the labeled saline solution. C.Use the saline solution and then relabel the bottle with the current date. D.Discard the saline solution and obtain a new unopened bottle.

D.Discard the saline solution and obtain a new unopened bottle.

The mental health nurse plans to discuss a client's depression with the health care provider in the emergency department. There are two clients sitting across from the emergency department desk. Which nursing action is best? A.Only refer to the client by gender. B.Identify the client only by age. C.Avoid using the client's name. D.Discuss the client another time.

D.Discuss the client another time. (Rationale: The best nursing action is to discuss the client another time. Confidentiality must be observed at all times, so the nurse should not discuss the client when the conversation can be overheard by others. Details can identify the client when referring to the client by gender or age, even when not using the client's name.)

Which intervention is most important to include in the plan of care for a client at high risk for the development of postoperative thrombus formation? A.Instruct in the use of the incentive spirometer. B.Elevate the head of the bed during all meals. C.Use aseptic technique to change the dressing. D.Encourage frequent ambulation in the hallway.

D.Encourage frequent ambulation in the hallway.

After receiving written and verbal instructions from a clinic nurse about a newly prescribed medication, a client asks the nurse what to do if questions arise about the medication after getting home. How should the nurse respond? A.Provide the client with a list of Internet sites that answer frequently asked questions about medications. B.Advise the client to obtain a current edition of a drug reference book from a local bookstore or library. C.Reassure the client that information about the medication is included in the written instructions. D.Encourage the client to call the clinic nurse or health care provider if any questions arise.

D.Encourage the client to call the clinic nurse or health care provider if any questions arise. (Rationale: To ensure safe medication use, the nurse should encourage the client to call the nurse or health care provider if any questions arise. Options A, B, and C may all include useful information, but these sources of information cannot evaluate the nature of the client's questions and the follow-up needed.)

The nurse is assisting a client to the bathroom. When the client is 5 feet from the bathroom door, he states, "I feel faint." Before the nurse can get the client to a chair, the client starts to fall. Which is the priority action for the nurse to take? A.Check the client's carotid pulse. B.Encourage the client to get to the toilet. C.In a loud voice, call for help. D.Gently lower the client to the floor.

D.Gently lower the client to the floor. (Rationale: Option D is the most prudent intervention and is the priority nursing action to prevent injury to the client and the nurse. Lowering the client to the floor should be done when the client cannot support his own weight. The client should be placed in a bed or chair only when sufficient help is available to prevent injury. Option A is important but should be done after the client is in a safe position. Because the client is not supporting himself, option B is impractical. Option C is likely to cause chaos on the unit and might alarm the other clients.)

Which nursing diagnosis has the highest priority when planning care for a client with an indwelling urinary catheter? A.Self-care deficit B.Functional incontinence C.Fluid volume deficit D.High risk for infection

D.High risk for infection

When the health care provider diagnoses metastatic cancer and recommends a gastrostomy for an older female client in stable condition, the son tells the nurse that his mother must not be told the reason for the surgery because she "can't handle" the cancer diagnosis. Which legal principle is the court most likely to uphold regarding this client's right to informed consent? A.The family can provide the consent required in this situation because the older adult is in no condition to make such decisions. B.Because the client is mentally incompetent, the son has the right to waive informed consent for her. C. The court will allow the health care provider to make the decision to withhold informed consent under therapeutic privilege. D.If informed consent is withheld from a client, health care providers could be found guilty of negligence.

D.If informed consent is withheld from a client, health care providers could be found guilty of negligence.

The nurse is preparing an older client for discharge. Which method is best for the nurse to use when evaluating the client's ability to perform a dressing change at home? A.Determine how the client feels about changing the dressing. B.Ask the client to describe the procedure in writing. C.Seek a family member's evaluation of the client's ability to change the dressing. D.Observe the client change the dressing unassisted.

D.Observe the client change the dressing unassisted. (Rationale: Observing the client directly will allow the nurse to determine if mastery of the skill has been obtained and provide an opportunity to affirm the skill. Option A may be therapeutic but will not provide an opportunity to evaluate the client's ability to perform the procedure. Option B may be threatening to an older client and will not determine his ability. Option C is not as effective as direct observation by the nurse.)

A client becomes angry while waiting for a supervised break to smoke a cigarette outside and states, "I want to go outside now and smoke. It takes forever to get anything done here!" Which intervention is best for the nurse to implement? A.Encourage the client to use a nicotine patch. B.Reassure the client that it is almost time for another break. C.Have the client leave the unit with another staff member. D.Review the schedule of outdoor breaks with the client.

D.Review the schedule of outdoor breaks with the client. (Rationale: The best nursing action is to review the schedule of outdoor breaks and provide concrete information about the schedule. Option A is contraindicated if the client wants to continue smoking. Option B is insufficient to encourage a trusting relationship with the client. Option C is preferential for this client only and is inconsistent with unit rules.)

Which serum laboratory value should the nurse monitor carefully for a client who has a nasogastric (NG) tube to suction for the past week? A.White blood cell count B. Albumin C.Calcium D.Sodium

D.Sodium

The nurse finds a client crying behind a locked bathroom door. The client will not open the door. Which action should the nurse implement first? A.Instruct an unlicensed assistive personnel (UAP) to stay and keep talking to the client. B.Sit quietly in the client's room until the client leaves the bathroom. C.Allow the client to cry alone and leave the client in the bathroom. D.Talk to the client and attempt to find out why the client is crying.

D.Talk to the client and attempt to find out why the client is crying. (Rationale: The nurse's first concern should be for the client's safety, so an immediate assessment of the client's situation is needed. Option A is incorrect; the nurse should implement the intervention. The nurse may offer to stay nearby after first assessing the situation more fully. Although option C may be correct, the nurse should determine if the client's safety is compromised and offer assistance, even if it is refused.)

A 20-year-old female client with a noticeable body odor has refused to shower for the last 3 days. She states, "I have been told that it is harmful to bathe during my period." Which action should the nurse take first? A.Accept and document the client's wish to refrain from bathing. B.Offer to give the client a bed bath, avoiding the perineal area. C.Obtain written brochures about menstruation to give to the client. D.Teach the importance of personal hygiene during menstruation with the client.

D.Teach the importance of personal hygiene during menstruation with the client. (Rationale: Because a shower is most beneficial for the client in terms of hygiene, the client should receive teaching first, respecting any personal beliefs such as cultural or spiritual values. After client teaching, the client may still choose option A or B. Brochures reinforce the teaching.)

A client has a nasogastric tube connected to low intermittent suction. When administering medications through the nasogastric tube, which action should the nurse do first? A.Clamp the nasogastric tube. B.Confirm placement of the tube. C.Use a syringe to instill the medications. D.Turn off the intermittent suction device.

D.Turn off the intermittent suction device. (Rationale: The nurse should first turn off the suction and then confirm placement of the tube in the stomach before instilling the medications. To prevent immediate removal of the instilled medications and allow absorption, the tube should be clamped for a period of time before reconnecting the suction.)

A postoperative client has three different PRN analgesics prescribed for different levels of pain. The nurse inadvertently administers a dose that is not within the prescribed parameters. What actions should the nurse take first? a. Access for side effects of the medication. b. Document the client's responses. c. Complete a medication error report. d. Determine if the pain was relieved.

a. Access for side effects of the medication. (This is a medication error. The first step in addressing a medication error is to access for any side effects of the medication on the patient. Certain analgesics may cause respiratory depression, so it is essential to monitor for vital sign changes or respiratory distress. Once noting the patient is stable, you may then contact the provider, document the response, and complete a medication error report.)

To prepare a client for the potential side effects of a newly prescribed medication, what action should the nurse implement? a. Assess the client for health alterations that may be impacted by the effects of the medication b. Teach the client how to administer the medication to promote the best absorption c. Administer a half dose and observe the client for side effects before administering a full dosage d. Encourage the client to drink plenty of fluids to promote effective drug distribution

a. Assess the client for health alterations that may be impacted by the effects of the medication (Before a new medication is given, an initial assessment should be completed to create a baseline for the patient; then the RN will be able to re-evaluate the patient and see if there have been any health alterations caused by the new medication. (B) this has nothing to do with potential side effects. (C) You should always administer a new medication as prescribed by the MD. (D) The amount of fluids the patient drinks will not affect the drug distribution in the body.)

The nurse receives a report that a client with an indwelling urinary catheter has an output of 150 mL, for the previous 6 hour shift. Which intervention should the nurse implement first? a. Check the drainage tubing for a kink b. Review the intake and output record. c. Notify the healthcare provider d. Give the client 8 oz of water to drink

a. Check the drainage tubing for a kink (The minimum amount of output a patient should have for one hour is 30mL. In 6 hours, it should at least be 180mL. Since it is under, the first thing to do is check the equipment, in this case the foley tubing! Kinks can prevent adequate output to be in the bag. (B) doesn't help the situation. (C) not necessary unless the tubing is not kinked, the patient has sufficient PO intake, and there are no issues with the foley. (D) giving water would be the last step if the patient is not fluid overloaded.)

The nurse in the emergency department observes a colleague viewing the electronic health record (EHR) of a client who holds an elected position in the community. The client is not a part of the colleague's assignment. Which action should the nurse implement? a. Communicate the colleague's actions to the unit charge nurse b. Send an email to facility administration reporting the action c. Write an anonymous complaint to a professional website d. Post a comment about the action on a staff discussion board

a. Communicate the colleague's actions to the unit charge nurse (Looking up patients who are not under your direct care is a HIPPA violation and may result in termination of employment, despite the patient's status in society or your curiosity. The first action to implement is to report to your Charge Nurse so he or she may report the incident to the appropriate chain of command.)

While suctioning a client's nasopharynx the nurse observes that the client's oxygen saturation remains at 94% which is the same reading obtained prior to starting the procedure. What action should the nurse take in response to this finding? a. Complete the intermittent suction of the nasopharynx. b. Reposition the pulse oximeter clip to obtain a new reading. c. Stop suctioning until the pulse oximeter reading is above 95%. d. Apply an oxygen mask over the client's nose and mouth

a. Complete the intermittent suction of the nasopharynx. (94% is perfect for suctioning! Only if you see the patient's O2 Stat drop below 90% is when you stop the suctioning and hyper-oxygenate the patient for a couple minutes. (B) not helpful. (C) it is ok to keep suctioning until you see a dip below 90% for O2. (D) not necessary at 94%.)

When evaluating the effectiveness of a client's nursing care, the nurse first reviews the expected outcomes identified in the plan of care. What action should the nurse take next? a. Determine if the expected outcomes were realistic b. Obtain current client data to compare with expected outcomes c. Modify the nursing interventions to achieve the client's goals d. Review related professional standards of care

a. Determine if the expected outcomes were realistic (Before you can determine if care is effective you need to be sure the original expected outcomes were realistic. (B) This step comes after (A). (C) Comes after evaluating if the goals are effective and obtaining data. (D) Not necessary in this case.)

he nurse observes a UAP positioning a newly admitted client who has a seizure disorder. The client is supine and the UAP is placing soft pillows along the side rails. Which action should the nurse implement? a. Instruct the UAP to obtain soft blankets to secure to the side rails instead of pillows b. Ensure that the UAP has placed pillows effectively to protect the client c. Ask the UAP to use some pillows to prop the client in a side-lying position d. Assume responsibility for placing the pillows while the UAP complete another task

a. Instruct the UAP to obtain soft blankets to secure to the side rails instead of pillows (In an ideal world, you would have seizure guard pads, but in the HESI world, you do not have it. SO! Pillows guarding the rails is dangerous and could suffocate your patient if they seizure and it falls and covers their face. Blankets secured to the side rails can be tied down to ensure they do not fall on the patient's face.)

The nurse is discharging an adult woman who was hospitalized for 6 days for treatment of pneumonia. While the nurse is reviewing the prescribed medications, the client appears anxious. What action is most important for the nurse to implement? a. Instruct the client to repeat the medication plan b. Encourage client to take a PRN antianxiety drug c. Provide written instructions that are easy to follow d. Include a family member in the teaching session

a. Instruct the client to repeat the medication plan (When a patient is anxious, they may not hear or retain what you are trying to teach them. So it is necessary to ask them repeat back your instructions so you can be sure they understood your teaching. This is called "teach back". (B) is not necessary and medication should not be a first line to rid a patient of anxiety. (C) while helpful, some patients may not be willing to read or if anxious, could not be focused enough to read. (D) while helpful, the patient may not always have a family member with them.)

A client is 2 days post-op from a thoracic surgery and is complaining of incisional pain. The client last received pain medication 2 hours ago. He is rating his pain a 5 on a 1-10 scale. After calling the provider, what is the nurse's next action? a. Instruct the client to use guided imagery and slow rhythmic breathing b. Provide at least 20 minutes of back massage and gentle effleurage c. Encourage the client to watch TV. d. Place a hot water circulation device, such as an Aqua K pad, to operative site

a. Instruct the client to use guided imagery and slow rhythmic breathing (If there are no other PRN pain medications available after an initial dose was given, it is most appropriate to call the provider, then switch to alternative pain management methods; like guided imagery and encouraging slow rhythmic breathing. (B) while massage may be helpful, it is inappropriate for incisional pain as it may open the sutures. (C) While distraction can help reduce pain, watching TV does not rid the patient of the pain. (D) NEVER place a circulation device on an operative site as it may open the sutures!)

10. At 0100 on a male client's second postoperative night, the client states he is unable to sleep and plans to read until feeling sleepy. What action should the nurse implement? a. Leave the room and close the door to the client's room b. Assess the appearance of the client's surgical dressing c. Bring the client a prescribed PRN sedative-hypnotic d. Discuss symptoms of sleep deprivation with the client

a. Leave the room and close the door to the client's room (Although the patient has stated he is unable to sleep, the patient has also stated he has a plan, "to read until feeling sleep", which implies the patient plans to sleep. Therefore, (D) is not necessary and (C ) is very unnecessary because it is a stronger sleep aid. Offering melatonin would be more appropriate, but since it is not an option, (A) is correct. (B) does not help the client sleep in any way. )

When performing blood pressure measurement to assess for orthostatic hypotension, which action should the nurse implement first? a. Position the client supine for a few minutes b. Assist the client to stand at the bedside c. Apply the blood pressure cuff securely d. Record the client's pulse rate and rhythm

a. Position the client supine for a few minutes (The first step to measure orthostatic hypotension is to lie the patient down in a supine position for 3-5 minutes, and measure their blood pressure before having the patient sit up. (B) comes after a measurement is made while sitting. (C) should be done anyways. (D) This step should be done after each position change.)

The nursing staff in the cardiovascular intensive care unit are creating a continuous quality improvement project on social media that addresses coronary artery disease (CAD). Which action should the nurse implement to protect client privacy? a. Remove identifying information of the clients who participated b. Recall that authored content may be legally discoverable c. Share material from credible, peer reviewed sources only d. Respect all copyright laws when adding website content

a. Remove identifying information of the clients who participated (Since the improvement project is being creating on a social media platform, it is imperative to have all names and patient identifiers removed to protect the client's identity and privacy. Any names posted, regardless of whether or not it is a social media platform or a peer-reviewed source is a HIPPA violation.)

The unlicensed assistive personnel (UAP) describes the appearance of the bowel movement of several clients. Which description warrants additional follow up by the nurse? (select all that applies). a. Solid with red streaks. b. Brown liquid c. Multiple hard pellets. d. Formed but soft. e. Tarry appearance

a. Solid with red streaks. Indicative of hemorrhoids or a lower GI Bleed. Not normal b. Brown liquid Indicative of an upset stomach or C.Diff. Not normal. c. Multiple hard pellets. Indicative of dehydration or a lack of fiber. Not normal. e. Tarry appearance. This is indicative of a lower GI bleed. Not normal.

To avoid nerve injury, what location should the nurse select to administer a 3 mL IM injection? a. Ventrogluteal b. Outer upper quadrant of the buttock c. Two inches below the acromion process d. Vastus lateralis

a. Ventrogluteal (2-3mL IM injections should always be made in the ventrogluteal area to minimize discomfort.)

The nurse attaches a pulse oximeter to a client's fingers and obtains an oxygen saturation reading of 91%. Which assessment finding most likely contributes to this reading? a. BP 142/88 mmHg b. 2+ edema of fingers and hands c. Radial pulse volume is +3 d. Capillary refill time is 2 seconds

b. 2+ edema of fingers and hands ((A) blood pressure has nothing to do with the patient's O2 saturation. (C) This means the patient's pulse is strong and appropriate, but does nothing to the O2 stat. (D) The patient is having appropriate blood flow and this has nothing to do with O2 stats. (B) is correct because when a patient has too much fluid in their body, they will have 2+ edema of fingers and hands, which means this fluid could also be in their lungs or sitting on their chest. Thus, decreasing their O2 stats.)

The nurse is teaching a client how to do active range of motion (ROM) exercises. To exercise the hinge joints, which action should the nurse instruct the client to perform? a. Tilt the pelvis forwards and backwards b. Bend the arm by flexing the ulnar to the humerus c. Turn the head to the right and left d. Extend the arm at the ide and rotate in circles

b. Bend the arm by flexing the ulnar to the humerus (Active range of motion is when the patient is completing the physical activity with physical assistance or manipulation from the nurse. The elbow is a hinge joint, as stated in the question, and should be exercised by bending the forearm (ulnar) to the humerus (bicep area).)

What assessment finding places a client at risk for problems associated with impaired skin integrity? a. Scattered macula of the face b. Capillary refill 5 seconds c. Smooth nail texture d. Absence of skin tenting

b. Capillary refill 5 seconds ("At risk for problems associated with impaired skin integrity" . "At risk" is the key here; (B) normal capillary refill time is less than 3 seconds. Anything greater means that the patient is not having effective blood flow to the fingers and can cause skin to start necrosis (die). (A)(C)(D) are all normal findings.)

During the admission assessment of a terminally ill male client that he is an agnostic. What is the best nursing action in response to this statement? a. Provide information about the hours and location of the chapel b. Document the statement of the client's spiritual assessment c. Invite the client to a healing service for people of all religions d. Offer to contact a spiritual advisor of the client's choice

b. Document the statement of the client's spiritual assessment

The nurse is providing wound care to a client with a stage 3 pressure ulcer that has a large amount of eschar. The wound care prescription states "clean the wound and then apply collagenase." Collagenase is a debriding agent. The prescription does not specify a cleaning method. Which technique should the nurse use to cleanse the pressure ulcer? a. Lightly coat the wound with povidone-iodine solution b. Irrigate the wound with sterile normal saline c. Flush the wound with sterile hydrogen peroxide d. Remove the eschar with a wet-to-dry dressing

b. Irrigate the wound with sterile normal saline (Eschar is a natural part of a healing wound and can only be removed by the physician or wound care nurse. Unless directly stated in wound care notes, the ONLY solution to be used to clean a wound is sterile normal saline. )

A CVA (stoke) patient goes into respiratory distress and is placed on a ventilator. The client's daughter arrives with a durable power of attorney, and a living will that indicates there should be no extraordinary life saving measures. What action should the nurse take? a. Refer to the risk manager b. Notify the healthcare provider c. Discontinue the ventilator d. Review the medical record

b. Notify the healthcare provider (A "DNR" or "FULL" code status change can only be completed by the Physician, and therefore requires you to first notify the Physician to put in an order for DNR to extubate the patient. If there is no specific order stating that the patient is a DNR, assume they are a full code! Only a living will or active Durable power of attorney can legally change the status of an unresponsive patient. (C ) can happen after (B) is completed and the MD changes the code status. (D) not this option because active legal documents will override possibly out of date medical records.)

Which assessment data reflects the need for the nurses to include the problem, "Risk for falls" in a client's plan of care? a. Recent serum hemoglobin level of 16g/dL b. Opioid analgesic received one hour ago c. Stooped posture with a steady gait d. Expressed feelings of depression

b. Opioid analgesic received one hour ago (Opioid analgesics can cause impaired balance, which puts the patient at a risk for falls. (A) is normal for males and slightly elevated for females but has nothing to do with increased fall risk. (C) "steady gait" negates the stooped posture and the patient is not prone to falls. (D) depression does not cause falls.)

Earlier this morning, an elderly Hispanic female was discharged to a LTC facility. The family members are now gathered in the hallway outside her room. What is the best action? a. Ask the family to wait in the cafeteria when the next of kin makes the necessary arrangements b. Provide space and privacy for the family to share their concerns about the client's discharge c. Ask the social worker to encourage the family to clear the hallway d. Explain to the family the client's need for privacy so that she can make independent decisions

b. Provide space and privacy for the family to share their concerns about the client's discharge (The problem here is that too many people in the hospital hallway can be a safety hazard, so the goal is to create a safe space for the family. Therefore (B) is the most appropriate answer. (A) is incorrect because it is moving the family into an open area which could violate the patient's privacy and the next of kin may not be the decision makers for the patient. (C) the social worker is busy, and it's not their job! (D) LTC infers the patient is elderly and may lack the ability to make independent decisions.)

The nurse is caring for a hospitalized client who was placed in restraints due to confusion. The family removes the restraints while they are with the client. When the family leaves, what action should the nurse take first? a. Apply the restraints to maintain the client's safety. b. Reassess the client to determine the need for continuing restraints. c. Document the time the family left and continue to monitor the client. d. Call the healthcare provider for a new prescription.

b. Reassess the client to determine the need for continuing restraints. (Restraints, whenever possible should be discontinued when appropriate to decrease the risk of skin breakdown or injury to the patient. Since the patient successfully had them off while the family was around, before you place them back on the patient, you should first see if they are even necessary to begin with. Therefore (A) is not appropriate until after the assessment is done. (C) is not necessary, but you should continue to monitor the patient. (D) is only necessary when you put the restraints back on the patient.)

The nurse observes an unlicensed assistive personnel (UAP) who is providing a total bed bath for a confused and lethargic client. The UAP is soaking the client's foot in a basin of warm water placed on the bed. What action should the nurse take? a. Remove the basin of water from the client's bed immediately b. Remind the UAP to dry between the client's toes completely c. Advise the UAP that this procedure is damaging to the skin d. Add skin cream to the basin of water while the foot is soaking

b. Remind the UAP to dry between the client's toes completely ( (B) is especially important in making sure the patient does not experience skin breakdown due to excessive moisture. Keeping the client's feet clean is necessary, but keeping the client's feet dry is extremely important in skin maintenance )

A female client's significant other has been at her bedside providing reassurances and support for the past 3 days, as desired by the client. The client's estranged husband arrives and demands that the significant other not be allowed to visit or be given condition updates. Which intervention should the nurse implement? a. Obtain a prescription from the healthcare provider regarding visitation privileges b. Request a consultation with the ethics committee for resolution of the situation c. Encourage the client to speak with her husband regarding his disruptive behavior d. Communicate the client's wishes to all members of the multidisciplinary team

b. Request a consultation with the ethics committee for resolution of the situation ((A) is not appropriate. (C) would cause excessive stress to the patient and the patient may not want to see her estranged husband at all. (D) while appropriate, it does not help calm the estranged husband or get him off the premises. (B) is most appropriate and professionals who are trained in ethical issues like this can take care of the situation.)

A male Native American presents to the clinic with complaints of frequent abdominal cramping and nausea. He states that he has chronic constipation and has not had a bowel movement in five days, despite trying several home remedies. Which intervention is most important for the nurse to implement? a. Evaluate the stool samples for presence of blood b. Assess for the presence of an impaction c. Determine what home remedies were used d. Obtain list of prescribed home medication

c. Determine what home remedies were used (The key aspect here is that the patient is "Native American" and that he has had constipation despite trying "several home remedies" (assume herbal). The most important thing to do is to have an exact list of the home remedies so that you can be sure that any prescribed medications don't interact with the remedies that are already in the patient's system.)

A policy requiring the removal of acrylic nails by all nursing personnel was implemented 6 months ago. Which assessment measure best determines if the intended outcome of the policy is being achieved? a. Number of staff induced injury b. Client satisfaction survey c. Health care-associated infection rate. d. Rate of needle-stick injuries by nurse.

c. Health care-associated infection rate. (Acrylic nails are known to carry loads of bacteria and increase the risk of healthcare-associated infections. Therefore, by banning the wearing of acrylic nails, you would expect the prevalence of healthcare-associated infections to decrease. Acrylic nails have nothing to do with staff induced injuries, needle-stick injuries, or patient satisfaction scores.)

A female UAP is assigned to take the vital signs of a client with pertussis for whom droplet precautions have been implemented. The UAP requests a change in assignment because she has not yet been fitted for a particulate filter mask. Which action should the nurse take? a. Advise the UAP to wear a standard face mask to take vital signs, and then get fitted for a filter mask before providing personal care b. Send the UAP to be fitted for a particulate filter mask immediately so she can provide care to this client c. Instruct the UAP that a standard mask is sufficient for the provision of care for the assigned client d. Before changing assignments, determine which staff members have fitted particulate filter masks

c. Instruct the UAP that a standard mask is sufficient for the provision of care for the assigned client (This is one of Elsevier's famous trick questions! You need to pay attention to the wording of the question and know your precaution measures and differences! The question asks about DROPLET precautions, not AIRBORNE precautions, so only a standard mask is required to enter this room. AIRBORNE precautions are for patients' with TB, meningitis, etc. Droplet are for patients with the flu, rhinovirus, etc. Be careful!!)

When measuring vital signs, the nurse observes that a client is using accessory neck muscles during respirations. What follow-up action should the nurse take first? a. Determine pulse pressure b. Auscultate heart sounds c. Measure oxygen saturation d. Check for neck vein distention

c. Measure oxygen saturation (Using accessory neck muscles during respirations is a serious sign of respiratory distress. The patient is a having a hard time breathing and as such, the first thing to do would be to measure oxygen saturation. (A) Pulse pressure is the difference between systolic and diastolicblood pressure. It is measured in millimeters of mercury (mmHg). It represents the force that the heart generates each time it contracts. (B) This has nothing to do with the heart. (D) Neck vein distention that is present is a sign of increased CVP (force on the aorta) and is not appropriate here.)

A client is admitted with a fever of unknown origin. To assess fever patterns, which intervention should the nurse implement? a. Document the client's circadian rhythms b. Assess for flushed, warm skin regularly c. Measure temperature at regular intervals d. Vary sites for temperature measurement

c. Measure temperature at regular intervals (In order to best assess when the fevers are coming, it would be best for the RN to measure the patient's temperature once an hour, in regular intervals so comparisons can be made with regards to the patient's activities or medications. (A) Circadian rhythms may have little effect on body temperature. (B) A patient could be flushed and not have a fever, so this is not a precise way to measure temperature. (D) Only one consistent site should or method should be used to measure temperature.)

A client with rheumatoid arthritis is experiencing chronic pain in both hands and wrists. Which information about the client is most important for the nurse to obtain when planning care? a. Amount of support provided by family members b. Measurement of pain using a scale of 0 to 10 c. The ability to perform ADLs d. Nonverbal behaviors exhibited when pain occurs

c. The ability to perform ADLs (The key words here are "Chronic pain" meaning the patient has dealt with this pain and lived with this pain for awhile. Therefore (B) is incorrect and more appropriate for acute pain or before you give pain meds. (A) is nice but has nothing to do with the patient's hands or wrists and the patient's plan of care. (D) while important, it is not the most important. With patients who have rheumatoid arthritis, ADLs are most important because if the patient has too much pain, they may not be able to complete their own basic care.)

While interviewing a client, the nurse records the assessment in the electronic health record. Which statement is most accurate regarding electronic documentation during an interview? a. The client's comfort level is increased when the nurse breaks eye contact to type notes into the record b. The interview process is enhanced with electronic documentation and allows the client to speak at a normal pace c. The nurse has limited ability to observe nonverbal communication while entering the assessment electronically d. Completing the electronic record during an interview is a legal obligation of the examining nurse

c. The nurse has limited ability to observe nonverbal communication while entering the assessment electronically (While looking at the computer, the RN may have limited ability to visualize the nonverbal communication from the patient. (A) you should be trying to initiate some, not constant eye contact with your patient while completing electronic documentation. (B) The patient can speak at a normal pace with or without HER. (D) while the electronic record during an interview becomes a legal document, it is an obligation for your job, not for legal matters.)

A female client with chronic back pain has been taking muscle relaxants and analgesics to manage the discomfort, but is now experiencing an acute episode of pain that is not relieved by this medication regime. The client tells the nurse that she does not want to have back surgery for a herniated intervertebral disk, and reports that she has found acupuncture effective in resolving past acute episodes. Which response is best for the nurse to provide? a. Surgery removes the disk and is the only treatment that can totally resolve the pain b. The medication regimen you previously used should be re-evaluated for dose adjustment c. Massage and hot pack treatments are less invasive and can provide temporary relief d. Acupuncture is a complementary therapy that is often effective for management of pain

d. Acupuncture is a complementary therapy that is often effective for management of pain

An older adult male client is admitted to the medical unit following a fall at home. When undressing him, the nurse notes that he is wearing an adult diaper and skin breakdown is obvious over his sacral area. What action should the nurse implement first? a. Establish a toileting schedule to decrease episodes of incontinence b. Complete a functional assessment of the client's self-care abilities c. Apply a barrier ointment to intact areas that may be exposed to moisture d. Determine the size and depth of skin breakdown over the sacral area

d. Determine the size and depth of skin breakdown over the sacral area (Before you can treat the skin breakdown, you must first measure and record the affected area so you can have something to compare after you begin your interventions/treatments. Then you can continue with (A)(B) and (C). )

What instruction should the nurse provide for an UAP caring for a client with MRSA who has an order for contact precautions? a. Do not allow visitors until precautions are discontinued b. Wear sterile gloves when handling the client's body fluid c. Have the client wear a mask whenever someone enters the room d. Don a gown and gloves when entering the return

d. Don a gown and gloves when entering the return (MRSA requires contact precautions; an isolation gown, gloves. (A) visitors can still come if they wear the appropriate PPE. (B) not necessary as sterile gloves are not needed for body fluid. (C) a standard mask is only needed for droplet precautions...minus this time of COVID.)

When assessing a male client, the nurse finds that he is fatigued, and is experiencing muscle weakness, leg cramps, and cardiac dysrhythmias. Based on these findings, the nurse plans to check the client's laboratory values to validate the existence of which? a. Hyperphosphatemia b. Hypocalcemia c. Hypermagnesemia d. Hypokalemia

d. Hypokalemia- (muscle weakness, leg cramps, and cardiac dysrhythmias. Normal range is 3.5- 5.0.)

A male client with unstable angina needs a cardiac catheterization, so the healthcare provider explains the risks and benefits of the procedure, and then leaves to set up for the procedure. When the nurse presents the consent form for signature, the client hesitates and asks how the wires will keep his heart going. Which action should the nurse take? a. Answer the client's specific questions with a short understandable explanation b. Postpone the procedure until the client understands the risks and benefits c. Call the client's next of kin and ask them to provide verbal consent d. Page the healthcare provider to return and provide additional explanation

d. Page the healthcare provider to return and provide additional explanation (A patient should not sign a consent if they do not completely understand the procedure, benefits and risks. Although you may have an understanding of the procedure, it is the Physician and physician ONLY who can review the process of the procedure and benefits/risks with the client. That task is out of your scope as an RN.)

In-home hospice care is arranged for a client with stage 4 lung cancer. While the palliative nurse is arranging for discharge, the client verbalizes concerns about pain. What action should the nurse implement? a. Explain the respiratory problems that can occur with morphine use. b. Teach family how to evaluate the effectiveness of analgesics. c. Recommend asking the healthcare professional for a patient-controlled analgesic (PCA) pump. d. Provide client with a schedule of around-the-clock prescribed analgesic use.

d. Provide client with a schedule of around-the-clock prescribed analgesic use. It is common for cancer patients to have recurrent pain and it is the patient's right to ask about aspects of their care, such as pain management. (A) Morphine is actually used for both pain and to help patient's breathe better, it is not an opioid and does not cause respiratory depression. (B) The family is not trained to do this and is not appropriate. Pain is subjective and only the patient can say if the analgesics are effective. (C) PCA pumps are given in extreme cases when pain cannot be controlled by a previously tried pain management schedule like (D).

UAP has lowered the head of the bed to change the lines for a client who is bedbound with a foley catheter and enteral tube feeds. Which change from the client warrants the most immediate intervention by the nurse? a. A feeding is infusing at 40 mL/hr through an enteral feeding tube b. The urine meter attached to the urinary drainage bag is completely full c. There is a large dependent loop in the client's urinary drainage tubing d. Purulent drainage is present around the insertion site of the feeding tube

d. Purulent drainage is present around the insertion site of the feeding tube (The wording here is key. If it were an NG tube feeding, then the patient must ALWAYS be at, at least a 30 degree angle. BUT because it says an enteral tube feeds (through the stomach), it is ok to lower the bed down all the way. That being said, purulent drainage (D) shows that the tube feeds are leaking and thus the nurse needs to pause the feeds and sit the patient up again. This is the most immediate need over the urine drainage bag being full or kinked.)

When entering the room of an adult male, the nurse finds that the client is very anxious. Before providing care, what action should the nurse take first? a. Divert the client's attention b. Call for additional help from staff c. Document the planned action d. Re-assess the client situation

d. Re-assess the client situation (When a patient is anxious, the first thing to do is re-assess them and find out why the are anxious. (A) can be helpful after finding out why they are anxious. (B) the patient is not a threat, no help is needed. (C) appropriate after you find out why they are anxious.)

The nurse is conducting an initial admission assessment for a woman who is Mexican- American and who is scheduled to deliver a baby by C-section in the next 24 hours. What should the nurse include in the assessment? a. Provider an interpreter to convey the meaning of words and messages in translation b. Commend the client for her patience after a long wait in the admission process c. Arrange for the hospital chaplain to visit the client during her hospital stay d. Rely on cultural norms as the basis for providing nursing care for this client

d. Rely on cultural norms as the basis for providing nursing care for this client (Whenever Elsevier points out the ethnicity or race of a patient, there are key cultural aspects you should look for in your answer (D). Don't assume (A) is correct because she could very well speak English!)

Which instruction should the nurse include in the discharge teaching plan for an adult client with hypernatremia? a. Monitor daily urine output volume b. Drink plenty of water whenever thirsty c. Use salt tablets for sodium content d. Review food labels for sodium content

d. Review food labels for sodium content (Hypernatremia is when a patient has a Sodium level that is too high, therefore it is most appropriate to teach the patient to check sodium levels on food labels before discharging. (A) While a high sodium level can decrease urine output, it is most important for the patient to be able to identify high sodium foods to decrease the risk of developing hypernatremia again. (B) Drinking too much water when thirsty can cause hyponatremia. (C) is incorrect because the patient is already at risk for developing high sodium levels again.)

A male client presents to the clinic stating that he has a high stress job and is having difficulty falling asleep at night. The client reports having a constant headache and is seeking medication to help him sleep. Which intervention should the nurse implement? a. Determine the client's sleep and activity pattern b. Obtain prescription for client to take when stressed c. Refer client for a sleep study and neurological follow-up d. Teach coping strategies to use when feeling stressed

d. Teach coping strategies to use when feeling stressed (Before medicating a patient, always consider other non-pharmaceutical therapeutic methods. In this case, the patient stated he has a high stress job that is CAUSING him to have difficulty sleeping. Therefore, teaching coping strategies to use when feeling stressed could help him sleep without the need for a sleeping pill. (A) is incorrect because the patient already said he is having difficulty sleeping, this would extra nice to know information. (B) save prescriptions for the last step if all other methods fail. (C) again, this is if the other methods fail first.)

he nurse is teaching a client about the use of the syringes and needles for home administration of medications. Which action by the client indicates an understanding of standard precautions? a. Remove needle before discarding used syringes b. Wear gloves to dispose of the needle and syringe c. Don a face mask before administering the medication d. Washes hands before handling the needle and syringe

d. Washes hands before handling the needle and syringe (this is standard precautions) (standard precautions; nuff said. If you don't know these, review them because it is the foundation for nursing!)


Ensembles d'études connexes

Human A&P Terms for the FINAL : Be able to define or describe each of the following. I will randomly choose 50 of these in which you will need to define

View Set

Control - Working with control charts

View Set

Series 63 Chapter 1 Continued Further

View Set

NURS intervention EXAM 2 PrepU & lecture quizzes

View Set

Chapter 2 - National Differences in Political, Economic, and Legal Systems

View Set